You are on page 1of 49

Kristf Mikls

Mtrixreprezentcik

A Dodekader forgscsoportjnak mtrixreprezentcija


Ahhoz, hogy a Pentagondodekader forgscsoportjnak mtrixreprezentcijt
fel tudjuk rni, elszr is el kell helyeznnk a Dodekadert a Descartes fle
koordintarendszerben. Ennek legjobb mdja az, ha a Dodekadert gy
helyezzk el, hogy hat le egy kocka hat lapjra essen, s az lek a koordinta
tengelyekkel prhuzamosan helyezkedjenek el. Ezt mutatja az 1. bra.

1. bra.
A kvetkez feladat a cscsok koordintinak meghatrozsa.
Ehhez vlasszuk a Dodekader lhosszt 1nek, azaz pl. az ab tvolsg = 1.
A koordintatengelyek legyenek a kvetkezk:
Az x tengely az ih szakasz felez merlegese, amely a kockalapra is merleges.
A y tengely az ek szakasz felez merlegese, amely a kockalapra is merleges.
A z tengely az ab szakasz felez merlegese, amely a kockalapra is merleges.
Krds, mekkora a kocka lhossza?
Ennek meghatrozshoz kell a 2. bra s a 3. bra.

2. bra.

3. bra.

Lthatjuk, hogy a + b + a = h, s a + b = 1, teht h = 1 + a.


Hasonl hromszgekbl addan a : b = (1 + a) : 1 = 1 + a = h,
b = 1 a miatt a : (1 a) = 1 + a, tszorozva ( 1 a)val :
a = (1 + a)(1 a) = 1 a2 , azaz a2 + a 1 = 0 , ennek megoldsa a =
h=1+a=

5 1
.
2

5 +1
. Ez ppen az aranymetszs szma. rtke h = 1.618033989
2

A tovbbiakban a h betvel mindig ezt a szmot jelljk (kivve amikor a h


cscsot emlegetjk).
A h szm tulajdonsgai a kvetkezk:
h2 = 1 + h, h3 = 1 + 2h, h4 = 2 + 3h, 1/h = h 1, 1/ h2 = 2 h.
A k tvolsg meghatrozsa:
k2 + 1/4 = h2 = 1 + h, k2 = 3/4 + h , k =

3
+h .
4

Ha a Dodekadert egy le fell nzzk, ezt ltjuk: (4. bra)

4. bra.
y = 1 + 2x lesz a Dodekadert befoglal kocka lhossza.
(y/2)2 + x2 = k2 , azaz (1/2 + x) 2 + x2 = 3/4 + h, azaz
1/4 + 2 x2 + x = 3/4 + h, teht 2 x2 + x h 1/2 = 0, ennek megoldsa
3

1
1 + 8(h + ) 1
8h + 5 1 1 + 2h 1 h
2
=
=
= ,
x=
4
4
4
2

mert 8h + 5 = (1 + 2h)2 = 1 + 4 h2 + 4h = 1 + 4 + 4h + 4h ,
felhasznltuk, hogy h2 = 1 + h.
y = 1 + 2x = 1 + h teht a kocka lhossza.
Ennek birtokban meg tudjuk adni a kockalapokra es cscsok koordintit.
a = ( 1/2, 0, (1 + h)/2)
b = ( 1/2, 0, (1 + h)/2)
e = (0, (1 + h)/2, 1/2)
k = (0, (1 + h)/2, 1/2)
h = ((1 + h)/2, 1/2, 0)
i = ((1 + h)/2, 1/2, 0)
A nem lthat oldalakon lev cscsok koordintja hasonl.

A kockacscsok irnyban lev cscsok a

1
1
1
,
,
egysgvektorok
3
3
3

irnyba esnek, hosszuk pedig annyi, amennyi a kockalapokon lev cscsok


tvolsga az origtl, minden cscs ugyanolyan messze van az origtl.

Ez a tvolsg z, s z2 = (y/2)2 + 1/4 , azaz z2 = (1 + h) 2 /4 + 1/4 = (2 + 3h +1)/4


1
3
1
1
h , ezzel szorozzuk a
,
,

2
3
3
3

h h h
egysgvektorokat. Kapjuk: , , , azaz pl.
2 2 2

azaz z2 = 3/4 (1 + h), teht z =

h h

f = , , ,
2 2 2
h h h

c = , , ,
2 2 2
h

h h

d = , , ,
2 2 2

l = , , ,
2 2 2
h h

n = , , , .
2 2 2
A cscsok birtokban most mr meg tudjuk hatrozni a forgat mtrixokat.
Azt kell csupn kifejezni, hogy az adott forgat mtrix melyik cscsot hova
viszi. Ezzel egy lineris egyenletrendszert kapunk, amit meg tudunk oldani.
Van mg egyszerbb t is azonban.
Ehhez azt kell tudni, hogy a forgat mtrix els oszlopa azt a vektort adja meg,
amibe a mtrix az x irny egysgvektort viszi. A mtrix msodik oszlopa azt a
vektort adja meg, amibe a mtrix az y irny egysgvektort viszi. Vgl a
mtrix harmadik oszlopa azt a vektort adja meg, amibe a mtrix a z irny
egysgvektort viszi. Ha teht ismerjk e vektorokat, akkor oszloponknt tudjuk
sszerakni a mtrixunkat!
Bizonyts:
a11

a 21
a
31
a11

a 21

a 31
a11

a 21

a 31

a12 a13 1 a11



a 22 a 23 0 = a 21 ,
a 32 a 33 0 a 31
a12 a13 0 a12

a 22 a 23 1 = a 22 ,
a 32 a 33 0 a 32
a12 a13 0 a13

a 22 a 23 0 = a 23 .
a 32 a 33 1 a 33

Egyszeren adhatk meg az x, az y s a z tengely krli 180oos forgatsok.


Az x tengely krl forgat X mtrix az x tengelyt helyben hagyja, az y t
y ra, a z t pedig z re cserli, ennek mtrixa teht
1 0 0

X = 0 1 0 .
0 0 1

Hasonlan
1

Y= 0
0

Z= 0
0

0 0

1 0 , s
0 1
0 0
1 0 .
0 1

X2 = Y2 = Z2 = Egysgmtrix.
A kockacscsok krli 120oos forgatsok mtrixai is egyszeren adhatk meg.
A c cscs krl forgat C mtrix pldul az x tengelyt az y tengelybe, az y
tengelyt a z tengelybe s a z tengelyt az x tengelybe viszi, mtrixa teht
0 0 1

C = 1 0 0 .
0 1 0

C2 ugyane cscs krl forgat csak visszafel:


0 1 0
C = 0 0 1
1 0 0

s termszetesen C3 = Egysgmtrix.
Hasonlan adhat meg az f cscs krl forgat F mtrix is:
F az x tengelyt a z tengelybe, az y tengelyt a x tengelybe s a z tengelyt a y
tengelybe viszi, mtrixa teht:
0 1 0

F = 0 0 1 .
1 0 0

F2 ugyane cscs krl forgat csak visszafel:


0 0 1

F = 1 0 0
0 1 0

s termszetesen F3 = Egysgmtrix.
A d, j, l, n cscsok krli forgats hasonlan adhat meg.

Ezzel azonban vget is rt az egyszeren megadhat forgatsok sora.


Az a, b cscsok krli hromfogs, az ab l krli ktfogs vagy az abfed lap
krli tfogs forgats mtrixnak megadsa mr komolyabb szmtsokat
ignyel. Ehhez mindenekeltt azt kell kiszmolni, hogy az egyes lek
felezpontja hova esik. Mivelhogy az x, y s z tengely is felezpontra, a hi, az
ek s az ab lek felezpontjra esik. (pontosabban az ek felezpont a ynak
felel meg). Az ab l felezpontja nem ms, mint az a s a b cscs koordintinak
a + b a1 + b1 a 2 + b2 a 3 + b3
,
,
=
, ahol a = (a1 ,a 2 ,a 3 ) az a
2
2
2
2
cscs koordinti s b = ( b1 , b2 , b3 ) a b cscs koordinti. Hasonlan kapjuk meg

szmtani kzepe, azaz

brmely ms l felezpontjt is. A felezpont ismeretben meghatrozhatjuk a


felezpont irnyba mutat egysgvektort is, ami azrt kell, mert az x, y, z
irnyba mutat egysgvektort a forgatmtrix szintn egysgvektorba fogja
vinni. Az egysgvektor kiszmtshoz meg kell hatrozni az lkzppontok
tvolsgt az origtl, ez minden cscsra ugyanannyi lesz, s mr kiszmoltuk,
ez az u =

1
1+ h
+x =
tvolsg, ami nem ms, mint a befoglal kocka
2
2

lhossznak fele.
Az egysgvektor kiszmtsa teht ez:
a + b a1 + b1 a 2 + b2 a 3 + b3 a1 + b1 a 2 + b 2 a 3 + b3
=
,
,
,
,
=
2u 2u
2u
2u 1 + h 1 + h 1 + h

Ellenrzskppen az ab szakaszfelez irnyba mutat egysgvektor ppen a z


irny egysgvektor kell legyen, azaz a (0, 0, 1) egysgvektor:
1 1+ h
1 1+ h
a = , 0,
s b = ,0,
, teht

2
2
2
2

a+b
1 1 1
1+ h 1+ h
=
,0 + 0,
+
= (0,0,1) valban.
2u 1 + h 2 2
2
2

Hatrozzuk meg az ef l irnyba mutat egysgvektort!

e = 0,

1+ h 1
, ,f=
2 2

h h h
2 , 2 , 2 , , teht

e+f
1
h 1+ h h 1 h
1 h 1 + 2h 1 + h 1
h 1
,
=
0 + ,
, + =
,
= , , .

2u 1 + h
2
2
2 2 2 1+ h 2
2
2 2h 2 2

Kzben ugye nem felejtettk el a h algebrai tulajdonsgait?


h2 = 1 + h, h3 = 1 + 2h, h4 = 2 + 3h, 1/h = h 1, 1/ h2 = 2 h.
A kapott vektor hossza termszetesen 1 kell legyen, azaz
2

2 h +1+ h +1 4
1 h 1
= =1 .
2h + 2 + 2 =
4
4

Most mr nekivghatunk a nagy kalandnak, kiszmolhatjuk a cscsok, lek s


lapok krl forgat mtrixokat!
Hatrozzuk meg pldul a b cscs krl az ramutatval ellenttes irnyba
forgat mtrixot! Legyen ez a B mtrix!
A B az ab let a bf lbe viszi, az fe let az fh lbe, s az fh let az ab lbe viszi.
Node azt nzzk meg, hogy hova viszi az x, y, z irny egysgvektort! ezzel
megkapjuk a mtrixunk els, msodik s harmadik oszlopt!
A most kvetkez szmtsokhoz mr nem kevs trltsi kpessg kell, nem
rt, ha van egy paprbl kszlt Dodekader modellnk, amit kedvnkre
forgathatunk ide oda! J ha a cscsait megjelljk a megfelel betkkel!
h h h 1 + h 1

,
Az x tengelyt, azaz a hi let az 1. brn mr nem lthat , , 0,
2 2
2 2 2

1 h 1

lbe viszi, az ennek irnyba mutat egysgvektor , , , ez teht a B


2h 2 2
mtrix els oszlopa.
h

h h 1 + h

, , 0 lbe viszi, melynek egysgvektora


Az y tengelyt a , ,
2
2
2 2 2
h 1 1
2 , 2 , 2h , ez lesz a B mtrix msodik oszlopa.

A z tengelyt (az ab l felezpontjt) az fb l felezpontjba viszi, azaz a


1 h
h h h 1 1 + h
1
2 , 2 , 2 2 ,0, 2 lbe, amelynek egysgvektora 2 , 2h , 2 .

Ez lesz teht a B mtrix harmadik oszlopa. Ezzel a B mtrix gy nz ki:

1
2h

h
B =
2

1
2

h
1
2
2
1
1

. Ez teht egy harmadrend mtrix kell hogy legyen.


2
2h

1
h
2h
2

Valban, szmoljuk ki B2et!


1
h2 1
+
2
4 4
4h
1 h 1
B2 =
4 4 4h
1 1 h
4h + 4 + 4

1 h 1
+ +
4 4 4h
h2 1 1
+ 2
4 4 4h
h 1 1
+
4 4h 4

1 1 h 1
h
+ +
4h 4 4
2
2h
h 1 1 h
1
+
=

4 4h 4
2
2

2
1
1
1 1
h

2 +
2h
4 4h
4 2

1
2
1
.
2h

h
2

Ltjuk, hogy B2 ppen a B transzponltja.


Ennek gy is kell lennie, B ugyanis ortogonlis mtrix, amelynek az inverze
ppen a transzponltja lesz. B egysgvektort egysgvektorba visz. A forgats
ugyanis nem vltoztatja meg a vektorok hosszt, csak az irnyt.
Kiszmolhatjuk, hogy B B2 ppen az egysgmtrix lesz.
Gyakorlskppen vgezzk is el nllan ezt a szmtst!
B a b cscsot helyben kell hogy hagyja. Valban,
1
1
2 2h

h
B 0 =
1+ h 2

1
2 2

h
1
1 1+ h
+
1
1

2
2
4h
4

2
2

1
1
h h

0 =

= 0 .
2
2h
4 4

1
h 1 + h 1 1 + 2h 1 + h
+
2h
2 2 4
4 2

Gyakorlskppen szmoljuk ki, hogy a B az a cscsot az f cscsba, az f cscsot


a c cscsba s a c cscsot az a cscsba viszi!
Na most a B mtrix birtokban, az X, Y, Z, C, F, stb. forgatsok segtsgvel a
Dodekader 60 elem forgscsoportjt, az A5 alternl csoportot teljes
egszben generlni tudjuk. Kt forgats egymsutnja is forgats, ezrt
alkotnak a forgatsok csoportot. Teht az X, Y, Z, C, F, s B mtrixok
szorozgatsval megkaphatjuk mind a 60 csoportelemet:

A Dodekadernek 12 lapja, 20 cscsa s 30 le van, ennek megfelelen


124/2 = 24 tfogs forgs, 202/2 = 20 hromfogs forgs s 30/2 = 15
ktfogs forgs van. s termszetesen a helybenhagysnak megfelel
egysgmtrix a 60-adik elem. gy 24 + 20 + 15 + 1 = 60 valban.
Ltezik egy jval komplikltabb t is egy tfogs mtrixelem
meghatrozshoz. Ezt most azrt prezentlom, hogy megmutassam, milyen
csodlatos dolog a matematika, ahogy a kezdeti koszbl kialakul a rend!
Buktassuk a Dodekadert az x tengely krl magunk fel gy, hogy egy lapja
ppen az xy skkal prhuzamos legyen! ezutn az xy skban hajtsunk vgre egy
72oos forgatst az ramutatval ellenttes irnyban, majd az x tengely krl
buktassuk vissza ugyanazzal a szggel! e 3 mvelet eredmnyekppen egy
tfogs forgats jn ltre.
Milyen szggel kell a Dodekadert buktatni? erre felel a 4. bra. Ott egy olyan
derkszg hromszget lthatunk, amelynek az tfogja k, a vzszintes
befogja y/2, a fggleges befogja pedig x hosszsg. A buktatst teht olyan
x
y
h
, cos = . Mivel x = ,
k
2k
2
3
h
1+ h
y = 1 + h, s k =
+ h , sin =
, cos =
,
2k
2k
4

szggel kell vgrehajtani, melyre sin =

szmszeren sin = 0.525731112, = 31.7174744o .


A forgat mtrixunk gy nz ki:

1
0
0
0
1
1+ h

U = 0 cos sin = 0

2k
0 sin cos

h
0
2k

.
2k
1 + h
2k

A k nem fejezhet ki h val racionlisan. Na itt jn be a kosz!


A 72oos forgatshoz sin 72 o s cos 72o kell, ezeket a 2. brbl lehet
k
h

meghatrozni, tudniillik sin 72 o = cos 18 o = , cos 72 o = sin 18 o =

k
cos 18 o = =
h

1
.
2h

3
3
h+
4 =
4 = h + 3 ( 2 h ) = 2h + 3 1 h 3 h = 1 + 1 h =

2
h
h
4
2
4
2 4

h+

10

= cos 18 o =

1 1+ 5
5+ 5
, na ez az az alak amire emlkeztem.
+
=
2
8
8

Ltjuk, hogy ktszeres gykvons van benne, ezen nem lehet segteni.
Mr csak abban a csodban remnykedhetnk, hogy ezek az irracionlis
kifejezsek a vgn valahogy eltnnek.
A 72oos forgat mtrixunk gy fog kinzni:
1
2h

k
V =
h

0
h

1
0 .
2h

0 1

A buktats, a 72oos forgats s a visszabuktats gy fog kinzni:


U1 V U , amely termszetesen fordtott sorrendben rtend, azaz
elszr U val forgatunk, majd Vvel, vgl U1gyel.
A mtrixszorzs asszociatv, teht tetszleges sorrendben szorozhatunk.
Szorozzuk ssze elszr az els kettt, majd az eredmnnyel szorozzuk a
harmadikat! Kapjuk:
1

2h
h
U 1V =
2

1
2

h
1

2
2
h
2h

3
2
2
3

h
h
h
h
h
h
h
, U 1VU =
+ 2 2 + 2 .

2
4k 2k
8k 4k
8k 4k
2
2
3
1 1+ h
1
h
h
h
h4

+
+

4k 2k
8k 2 4k 2 8k 2 4k 2
2

A nevezkben mindentt k2 szerepel, ami mr racionlis kifejezs!


k2 = h +

3
, mi ennek a reciproka? Mondjuk a + bh. Szorozzuk meg vele:
4

11

(h +

3
)(a + bh) = 1 kell legyen.
4

ah +

3
3
a + b + bh + bh =1.
4
4

a+b+

3
7
b = 0, a = b .
4
4

3
a + b = 1,
4

3 7
( )b + b = 1,
4 4

(1

21
)b = 1,
16

b=

16
.
5

3
16 21
28
a = 1+
=
,a=
.
4
5
5
5

Teht

1
7 4
= h , ezt kell betenni a mtrixunkba:
2
8k 10 10

1
2h

h
U 1VU =
2

1
2

h
2

7 4
h (
(1 + 2h + 2 + 2h ) 10
10

7
4
1 h + 2 + 4h h .
10 10

7
4
h + 4 + 6h h
10 10

1
2

7
4
1 h + 2 + 4h h
10 10

Vgezzk el a kijellt szorzsokat!


7 4
7 4
h = (3 + 4h ) h =
(1 + 2h + 2 + 2h ) 10
10
10 10
=

21 12
28 16 16
5 1
h+ h h = = ,
10 10
10
10 10
10 2

7 4
7 4
h = (1 + 3h ) h =
(1 h + 2 + 4h ) 10
10
10 10
=

7 4
21 12 12
5 5
1 1
1
h+ h h= + h= + h= ,
10 10
10
10 10
10 10
2 2
2h

7 4
7 4
h = ( 4 + 7h ) h =
(h + 4 + 6h ) 10
10
10 10

12

28 49
16
28 28
5
1
h
+ h h h = h = h = .
10 10
10
10 10
10
2
2

Ezzel a mtrixunk gy alakul teht:


1

2h
h
U 1VU =
2

1
2

h 1
2 2
1
1
.
2 2h
1
h
2h 2

Nevezzk ezt a mtrixot Pnek! Ez teht egy tdrend forgats.


Ellenrzskppen szmtsuk ki a mtrix determinnst!
Ennek egynek kell lennie.
Det P =

1 h 1 h h2 1 1 1 1

+ + +
+ =
2h 4 4h 2 2 4 4h 2 4 4

1
1
8
(h 1)(h 2 + h) + h(1 + h + h 1) + 2 ) = (2 + 2h 2h 2h + 2 + 2 + 2h + 2) = = 1
(
8
8
8

Szmoljuk ki P hatvnyait! Az eredmny:


h

2
1
P2 =
2

2h

h
1
1 1
1
1
h
1

2 2h
2
2h
2
2
2
2h
1 h 3 1
1
h 4 h 1
1
, P =
, P =

,
2h 2
2
2h
2
2
2
2h

h
1
h
1
1 h
1
1
2h
2 2h 2
2
2
2
2

Ltjuk, hogy P4 ppen P transzponltja, teht inverze, s nem is csaldunk,


P4 ppen az egysgmtrix lesz. Ezzel igazoltuk, hogy P tdrend.
P az a cscsot a b cscsba, a b cscsot a c cscsba viszi, s az e cscsot a h
cscsba viszi. ezzel 3 egyenletrendszert kapok, melybl P meghatrozhat!

13

Nzzk pl. a Pa = b cscstranszformcit!


1

2h
h
Pa =
2

1
2

2h
h
Pb =
2

1
2

1 1+ h
h 1
+
1
1
2 2
4
4h
2
2

1
1
h
h

0 = +

= 0 =b:
2 2h
4
4

1+ h
+
1
h

1
h
1 + 1 + 2h
2
2

4
2h 2
4

1 1+ h h
h 1
+
1
2 2
4 2
4h
2
h h h
1
1
0 =
+

=2=c
2 2h
4
4


1
h 1 + h 1 1 + 2h h
+
2h 2 2 4
4 2

Ebbl a kvetkez egyenletrendszer lesz:


1
1+ h
1
p11 +
p13 = ,
2
2
2
1
1+ h
h
p11 +
p13 = .
2
2
2

Az egyenletrendszer megoldsa: p11 =

1
1
, p13 = , s tnyleg annyi!
2h
2

Hasonlkppen lehet a tbbi mtrixelemet kiszmolni.


A P forgats az tszg kzppontjt fixen hagyja.
Az tszg kzppontjt pedig gy kapom meg, hogy a z irny egysgvektort
szggel buktatom az x tengely krl. gy az tszg tengelyt a
h 1+ h
,
egysgvektor hatrozza meg. Teht Pp = p kell legyen.
2k 2k

p = 0,

h h 1 1+ h 1 h
1 1+ h 1 h h 1+ h h 1+ h
Pp = +
, +
, +
,
= 0,

2 2k 2 2k 2 2k 2h 2k 2h 2k 2 2k 2k 2k

14

Valban a p vektort kaptuk teht.


Hatrozzuk meg azt a forgatst, amely az a b f e d a cscsokat
viszi egymsba! J, ha kznl van egy j paprmodell.
Nevezzk ezt a mtrixot Qnak! Teht Qa = b, Qb = f, Qf = e, stb.
Q a hi let a kl lbe viszi, teht az x tengelyt az
1 h h h 1
1+ h 1 1 h 1
, , +
0,
, = , ,
1+ h 2 2 2 1+ h
2
2 2h 2 2
vektorba viszi, ez lesz teht Q els oszlopa.
Q az ek let a dg lbe viszi, teht a y tengelyt az
1 h h h 1 1+ h 1 h 1
, , +

, ,0 = , ,0
1 + h 2 2 2 1 + h 2
2 2 2
vektorba viszi, teht ennek 1 szerese lesz Q msodik oszlopa.
Q az ab let a bf lbe viszi, teht a z tengelyt az
1 h h h 1 1 1+ h 1 1 h
, , +
,0,
= , ,
1 + h 2 2 2 1 + h 2
2 2 2h 2
vektorba viszi, ez lesz teht Q harmadik oszlopa. Kaptuk teht:
1

2h
h
Q =
2

1
2

h
1
2
2
1
1
.

2
2h
1
h

2h
2

Meglv mtrixaink szorozgatsval pedig a Dodekader forgscsoportjnak


mind a 60 elemt megkaphatjuk. Szorgalmi feladat: szmoljunk ki minl
tbbet! Prbljuk meg feltrkpezni a 60 elem csoportot!

15

Harmadfok egyenletek
Sok olyan problma ltezik, amely harmadfok egyenlethez vezet.
Elszr vegynk egy egyszerbb s ismertebb esetet, a Fibonacci sorozatot!
Ennek Maple 7 programja: (A tovbbiakban piros szn = Maple 7 program)
f[1]:=1:f[2]:=1:for n from 3 to 30 do f[n]:=f[n-1]+f[n-2] od:
seq(f[n],n=1..30);
1, 1, 2, 3, 5, 8, 13, 21, 34, 55, 89, 144, 233, 377, 610, 987,
1597, 2584, 4181, 6765, 10946, 17711, 28657, 46368, 75025,
121393, 196418, 317811, 514229, 832040

Amint ltjuk, a Fibonacci sorozat elemeit gy kpezzk, hogy az elz kettt


sszeadjuk, pl. 13 = 5 + 8, 144 = 55 + 89, stb.
Kpezzk kt egymst kvet elem hnyadost, azaz az

f n + 1
hnyadost!
f n

5
1
2
3
Kapjuk: = 1 , = 2 , = 1.5 , = 1.66666... , s gy tovbb haladva kapom az
1
1
2
3
1, 2, 1.5, 1.6666666, 1.6, 1.625, 1.615384615, 1.619047619, 1.617647059 . . .
szmsorozatot. Jl ltszik hogy ezek a szmok mind jobban megkzeltenek
egy hatrrtket, ami 1.618033989 Vajon mi ez a szm?
Ha n elg nagy, akkor

f n + 1
mr elg jl megkzelti ezt a szmot, pl.
f n

f n + 1
832040
= 1.618033989 . Ekkor mondhatom azt, hogy
= x, s
514229
f n
f n 1 1
= . f [n+1] = f [n] + f [n 1] , s osszuk ezt el f [n] nel!
x
f n

16

1
Kapjuk: x = 1 + . Szorozzuk meg x szel: x 2 = x + 1 , azaz x 2 x 1 = 0 .
x

Ennek a msodfok egyenletnek a megoldsa: x =

1+ 5
.
2

Ez teht a titokzatos 1.618033989 jelentse!


Ezt a szmot gy nevezik, hogy aranymetszs. eljn az tg csillagnl,
s a Pentagondodekadernl is. A Rgiek a szpsg megtestestjt lttk
benne. Felbukkan az ptszetben, a szobrszatban, st a zenben is.
Csinljunk most egy msik sorozatot, ami hasonl a Fibonaccihoz:
a[1]:=1:a[2]:=1:a[3]:=1:for n from 4 to 40 do
a[n]:=a[n-2]+a[n-3] od:
seq(a[n],n=1..40);
1, 1, 1, 2, 2, 3, 4, 5, 7, 9, 12, 16, 21, 28, 37, 49, 65, 86,
114, 151, 200, 265, 351, 465, 616, 816, 1081, 1432, 1897,
2513, 3329, 4410, 5842, 7739, 10252, 13581, 17991, 23833,
31572, 41824

Itt nem a kt kzvetlen megelzt adjuk ssze, hanem az eggyel elbbieket.


Pl. 28 = 12 + 16, 86 = 37 + 49, 114 = 49 + 65, stb.
Nzzk meg, mi itt az egymst kvet szmok arnya!
seq(evalf(a[n+1]/a[n]),n=1..39);
1.,1., 2., 1., 1.500000000, 1.333333333, 1.250000000,
1.400000000, 1.285714286, 1.333333333, 1.333333333,
1.312500000, 1.333333333, 1.321428571, 1.324324324,
1.326530612, 1.323076923, 1.325581395, 1.324561404,
1.324503311, 1.325000000, 1.324528302, 1.324786325,
1.324731183, 1.324675325, 1.324754902, 1.324699352,
1.324720670, 1.324723247, 1.324711500, 1.324722139,
1.324716553, 1.324717562, 1.324718956, 1.324717128,
1.324718357, 1.324717915, 1.324717828, 1.324718105

Ltjuk, hogy itt is van egy hatrrtk, ami 1.324717 krli.

17

Vajon hogy kapjuk meg ezt a szmot? Legyen most is

a n + 1
= x, s
a n

a n 1 1
a n 2 1
= , valamint
= 2!
x
a n
a n
x
Ekkor az a [n+1] = a [n 1] + a [n 2] egyenletbl a[n]nel val oszts utn
ez lesz : x =

1 1
+ 2 , amibl x2 tel val szorzs utn ez lesz: x 3 = x + 1 .
x x

Ez bizony mr harmadfok a javbl! Hogyan kell az ilyet megoldani?


Keressk a megoldst sszeg alakban: x = u + v .
Ekkor x3 = (u + v)3 = u3 + v3 + 3 u2 v + 3 u v2 = u3 + v3 + 3 u v (u + v).
Legyen u = 3 a + b , s v = 3 a b !
Ekkor u3 + v3 = a + b + a b = 2 a,
s 3 u v (u + v) = 3 3 a 2 b 2 (u + v) = 3 3 a 2 b 2 x
Teht x3 = 2 a + 3 3 a 2 b 2 x lesz az egyenletnk.
sszehasonltva az eredetivel, azt ltjuk, hogy 2 a = 1,
1
a= ,
2

a 2 b2 =

3 3 a 2 b 2 = 1.

1 1
23
1 1 2

=
= b , b =
.
27 4
4 27
108

1
23 3 1
23
x=3 +
+
. Szp!
2 108
2 108
Kiszmolva

23
= 0.461479103 , x = 0.986991206 + 0.337726751
108

x = 1.324717957. x3 = 2.324717957 = 1 + x valban!


Meg tudtuk teht oldani a harmadfok egyenletet, ezzel az egyszer trkkel!

18

ltalnosabb esetben x3 = 3 p x + 2 q,
ekkor is x = u + v, u = 3 a + b , s v = 3 a b , x3 = 2 a + 3 3 a 2 b 2 x ,
amibl q = a,

p = 3 a 2 b2 ,

amibl b = q 2 p3 , s

x = 3 q + q 2 p3 + 3 q q2 p3 .
Node mi trtnik, ha q2 p3 negatv? Akkor a gykkifejezs kpzetes lesz, s
komplex szmbl kell kbgykt vonni! s ha mr a komplex szmok szba
jttek, tudjuk hogy a harmadfok egyenletnek mindig 3 gyke van, ebbl
legalbb az egyik vals. Egsz pontosan 3 eset van:
Ha a q2 p3 pozitv, akkor egy vals s kt konjuglt komplex gyk van.
Ha a q2 p3 negatv, akkor hrom vals gyk van.
Ha a q2 p3 nulla, akkor hrom vals gyk van, melybl kett megegyezik.
Prbljuk most meghatrozni, mennyi sin (10o) !
Ha gppel szmoljuk ki, akkor sin (10o) = 0.173648177
a 10o a 30o egyharmada, s mint tudjuk, a szgharmadols ppen azrt
nem szerkeszthet meg krzvel s vonalzval, mert harmadfok egyenletre
vezet! sin (30o) = 0.5 , ha x = 10o , akkor sin (30o) = sin (3 x).
sin ( 3 x) = sin (2 x + x) = sin (2 x) cos (x) + cos (2 x) sin (x) =
= 2 sin (x) cos (x) cos (x) + (cos2 (x) sin2 (x)) sin (x) =
= 2 sin (x) cos2 (x) + (1 sin2 (x) sin2 (x)) sin (x) =
= sin (x) (2(1 sin2 (x)) + 1 2 sin2 (x)) = sin (x) (3 4 sin2 (x)) .

19

Ha s = sin (x), akkor sin (3 x) = 3 s 4 s3 = 0.5 az egyenletnk.


3 1
1
1
1
1
1 3 1
1
1
s3 = s , p = , q = , s = 3 +

.
4 8
4
16
16
256 64
16
256 64

Lthatjuk, hogy a ngyzetgyk alatti kifejezs, teht a diszkriminns negatv,


ezrt 3 vals gykt kell kapnunk. az m, de hogy vonunk a komplex szmbl
kbgykt? Nos, erre van egy mdszer.
A komplex szmot trigonometrikus alakba rjuk, s ezutn mr knny a
dolgunk. Ha a komplex szm trigonometrikus alakja z = r (cos () + i sin ()),

akkor a z kbgyke ilyen lesz: 3 z = 3 r (cos ( ) + i sin ( )).


3
3
Ha teht a diszkriminns negatv, akkor az x gy nz ki:
x = 3 a + ib + 3 a ib .

Most azt mutatom meg, hogyha 3 a + ib = c + id , akkor 3 a ib = c id !


Valban, emeljk kbre mindkt kifejezst:
a + ib = (c + id)3 = c3 id3 + 3 c2 id 3 c d2 , innen
a = c3 3 c d2 , b = 3 c2 d d3 .
(c id)3 = c3 + id3 3 c2 id 3 c d2 , s ez valban a ib,
az elbbi a, b kifejezssel!
Akkor pedig x = c + id + c id = 2 c lesz az egyik vals gyknk!
Mi lesz a msik kt vals gyk?
Ha a komplex szm trigonometrikus alakja a + ib = z = r (cos () + i sin ()),

akkor a z kbgyke ilyen lesz: 3 z = 3 r (cos ( ) + i sin ( )) = c + id.


3
3
20

Node + 360o = s + 720o = miatt megolds lesz a


3

z = 3 r (cos ( +120 o) + i sin ( +120 o)) s a


3
3

z = 3 r (cos ( +240 o) + i sin ( +240 o)) is!


3
3

Ez lesz teht a msik kt gyk!


Matematikailag ez azt jelenti, hogy 3 z mellett megolds a
3

z (cos (120 o) + i sin (120 o)) s a

z (cos (240 o) + i sin (240 o)) is!

cos (120 o) + i sin (120 o)= s cos (120 o) + i sin (120 o) = 2 jellssel
a hrom megolds gy nz ki: 3 z , 3 z , s 3 z 2 .
Az az gynevezett harmadik egysggyk, s az albbi egyenletnek
engedelmeskedik: 2 + + 1 = 0.
Ha most berjuk, hogy cos (120 o) =
cos (240 o) =

3
1
, valamint
s sin (120 o) =
2
2

1
3
s sin (240 o) = , akkor kapjuk:
2
2

1
3
1
3
= +i
s 2 = i .
2
2
2
2
Lthatjuk, hogy + 2 = 1 , s 2 = i 3 .
Most azt mutatom, meg, hogyha az x1 = u + v megoldsa az
x3 = 3 p x + 2 q egyenletnek, akkor az x2 = u + 2v s az x3 = 2u + v
is megoldsa lesz ugyanannak az egyenletnek!

21

x1 3 = (u + v) 3 = u3 + v3 + 3 u2 v + 3 u v2 = u3 + v3 + 3 u v (u + v).
Teht x1 3 = u3 + v3 + 3 u v x1 .
x2 3 = (u + 2v) 3 = 3u3 + 6v3 + 3 4u2 v + 3 5u v2 =
= u3 + v3 + 3 u2 v + 3 2u v2 = u3 + v3 + 3 u v (u + 2v).
Figyelembevettk, hogy 3 = 1 !
Teht x2 3 = u3 + v3 + 3 u v x2 .
x3 3 = (2u + v) 3 = 6u3 + 3v3 + 3 5u2 v + 3 4u v2 =
= u3 + v3 + 3 2u2 v + 3 u v2 = u3 + v3 + 3 u v (2u + v).
Teht x3 3 = u3 + v3 + 3 u v x3 .
Amikor u = c + id s v = c id volt, akkor az els vals gyk
x1 = c + id + c id = 2c. Mi a msik kt vals gyk?
1
3
1
3
x2 = (c + id) + 2 (c id) = ( + i ) (c + id) + ( i ) (c id):
2
2
2
2
x2 = c 3 d , s
1
3
1
3
x3 = 2 (c + id) + (c id) = ( i ) (c + id) + ( + i ) (c id):
2
2
2
2
x3 = c + 3 d .
Most mr visszatrhetnk a sin (10o) meghatroz egyenlethez!
3 1
1
1
1
1
1 3 1
1
1

s3 = s , p = , q = , s = 3 +
4 8
4
16
16
256 64
16
256 64

s=3

1
3 3 1
3
+i
+ i
.
16 16
16 16

22

r2 =

1
3
1
+
= ,
256 256 64

cos() =

1
r= ,
8

1
r= .
2

1
1
= , = 120o,
16 r
2

c = 3 r cos (40 o) =
= sin(50o).

= 40 o .
3

1
cos (40 o), x1 = 2 c = cos (40 o) = 0.766044443 =
2

Ht ez elg meglep! Nem sin (10o)ot kaptunk, ahogy vrtuk!


Node nzzk tovbb!
d = 3 r sin (40 o) =

1
sin (40 o), s ezzel
2

1
3
sin (40 o) = 0.93969262 = sin (70 o)!
x2 = c 3 d = cos (40 o)
2
2

Ez mg mindig nem a vrt sin (10 o) !


1
3
sin (40 o) = 0.173648177 = sin (10 o) !
x3 = c + 3 d = cos (40 o) +
2
2

Na vgre, csak kijtt!


Ltjuk, hogy a megolds nem mindig trivilis!
Mirt ppen ezek a szgek jttek ki?
Az volt a kiktsnk, hogy sin (3 x) = 0.5 legyen. Milyen szgekre
teljesl ez? 3 x = 30o , 150o , 390o , 510o , 750o , 870o , stb., ezek harmada:
x = 10o , 50o , 130o , 170o , 250o , 290o ,stb., ezek szinusza:
0.173648177, 0.766044443, 0.766044443, 0.173648177, 0.93969262,
0.93969262, stb., teht a 3 gyknk addik ki gy valban.

23

Most nzznk egy olyan esetet, ahol mr tudjuk a gykket!


Legyen az egyenletnk ez: (x 1)(x 2)(x + 3) = 0 !
Ennek szemmel lthatan 3 vals gyke van: x1 = 1, x2 = 2, x3 = 3 !
Szorozzuk most ssze a 3 zrjeles kifejezst! Kapjuk:
x3 + ( 1 2 + 3) x2 + (12 13 23) x + 123 = 0 , azaz
x3 7 x + 6 = 0 .
Ltjuk, hogy az x2 es tag egytthatja nulla, megvallom, direkt gy
vettem fel a 3 gykt!
Na az ember azt hinn, hogy ilyen egyszer esetben a megolds is egyszer
lesz. De mint ltni fogjuk, ez nem gy van!
x3 7 x + 6 = 0 .
Csak gy tesztelskppen x=1 re ez 1 7 + 6 = 0,
x = 2 re ez 8 14 + 6 = 0 s x = 3 ra 27 + 21 + 6 = 0 t ad,
teht jl rtuk fel az egyenletet. Sosem rt az vatossg!
7
p= ,q = 3 ,
3

x1 = 3 3 + 9

343 3
343 3
10 3
10
+ 3 9
= 3 + i
+ 3 i
.
27
27
27
27

Na ez szp, de hogyan vgezzk el?


343 3
7
100 343
3
, r=
, cos() =
=
, r=
= 0.841697576 ,
27
27
27
3
343
27
10
= 0.539949247 .
= 147.3198161 . sin() =
343

r2 = 9 +

24


7
x1 = 2 c = 2 3 r cos( ) = 2 cos(49.10660537) = 2 !
3
3
Na megvan akkor az egyik gyknk!
Ebbl rgtn kiderl, hogy akkor

3
= 0.65465367 !
cos( ) = cos(49.10660537) =
3
7
Akkor pedig c = 1 !

7
7
sin(49.10660537) =
0.755928946 = 1.154700538 .
d = 3 r sin( ) =
3
3
3
x2 = c 3 d = 1 1.732050808 1.154700538 = 1 2 = 3.
x3 = c + 3 d = 1 + 1.732050808 1.154700538 = 1 + 2 = 1.
Ht, elllt a hrom gyknk.
3 d = 2 miatt d =
3

a + b = 3 3 + i

2
, s ebbl lthatan
3

10
2
= 1+ i
= c + i d lesz a kbgykvons eredmnye!
27
3

Valban, emeljk kbre ezt a kifejezst:


3

2
2
4
8
18 8
10
(c + i d) = 1 + i = 1 + 3 i 3 i
= 3 + i
= 3 + i
.
3
3
3
27
27
27

Abban a szerencss helyzetben vagyunk teht, hogy a komplex szmbl


kbgykt tudtunk vonni, s az eredmnyt ki tudtuk fejezni gykkifejezssel!
De nem mindig van ez gy, a legtbbszr knytelenek vagyunk a
trigonometrikus alakot hasznlni.

25

Lttuk, hogy az egyik gyk 2 c, a msik gyk c 3 d , a harmadik gyk


c + 3 d alak.
2 c = 2, c 3 d = 3 , c + 3 d = 1 volt a szereposzts.
Vajon ez az egyetlen lehetsges vlaszts?
1
1
5
.
Legyen pl. 2 c = 1, c 3 d = 2 ! Akkor c = , 3 d = 2 , d =
2
2
2 3

1
5
6
c + 3 d = 3
= = 3 lesz! Kijtt a harmadik gyk!
2
2
2 3
sszesen hatfle szerepkioszts lehetsges, s mind a hat j!
A gykk s egytthatk sszefggsei:
x1 + x2 + x3 = 2 c c 3 d c + 3 d = 0, ez a msodfok tag egytthatja!
x1 x2 + x1 x3 + x2 x3 =
= 2 c ( c 3 d ) + 2 c ( c + 3 d ) + ( c 3 d ) ( c + 3 d ) =
= 3 c2 3 d2 = 3 3 r 2 = 3 p , az elsfok tag egytthatja!
x1 x2 x3 = 2 c ( c 3 d ) ( c + 3 d ) = 2 (c3 3 c d2 ) =
= 2 a = 2 q , a nulladfok tag egytthatja!
Most nzznk egy harmadfok egyenletre vezet pldt:
hatrozzuk meg az egysgnyi oldal htszg kt tljnak a hosszt!
Abbl, hogy a feladat harmadfok egyenletre vezet, kiderl, hogy a htszget
nem lehet krzvel s vonalzval megszerkeszteni!

26

Teht jjjn a szablyos htszg!

1. bra

2. bra

A htszg oldala 1, a hossz tl hossza 1 + 1 + x = 2 + x, vkony piros


vonal, Kt hossz tl ltal bezrt szg =
a htszg kt oldala ltal bezrt szg 5 =
a hossz tl hossza h =

2sin( )
2

180
= 25.71428571 ,
7

900
= 128.57142857 ,
7

= 2.246979604 = 2 + x , ebbl

x = 0.246979604 addik.
Ez az x, mint ltni fogjuk, egy harmadfok egyenletnek tesz eleget.
A rvid tl hossza egy koszinuszttelbl addik:
a 2 = 1 + 1 2 cos(5) = 3.246979604 , vegyk szre, hogy ez ppen x +3 !
a = 1.801937736.
Most nzzk meg, milyen egyenlet hatrozza meg xet!
A 2. brn lthat kk hasonl hromszgekbl:
x x +1
x
=
, amibl y =
.
y
1
x +1
A rzsaszn hromszgekbl:
27

1+ x + y 2 + x
=
, ebbe betve y t:
x + 2y
1
x
2
1 + x = 2 + x , azaz x + 2x + 1 + x = 2 + x , tszorozva:
x
x 2 + x + 2x
x+2
1+ x

1+ x +

x 2 + 3x + 1 = (2 + x)(x 2 + 3x) = 2x 2 + 6x + x 3 + 3x 2 = x 3 + 5x 2 + 6x .
Rendezve:
x3 + 4x 2 + 3x 1 = 0 .
Ez lesz teht az egyenletnk.
Kellemetlen vonsa, hogy van benne x2es tag is.
4
ennek kikszblsre vezessk be az u j vltozt! x = u .
3
3

4
4
4
u + 4 u + 3 u 1 = 0 .
3
3
3

4
16 64
16 8
4
u 3 3u 2 + 3u + 4 u 2 + u + 3 u 1 = 0 .
3
9 27
9 3
3
u3 +

16
64 64 32
u + u + 3u 5 = 0 .
3
27 9 3

7
7
u3 u = 0 .
3
27
7
7
1323
p = , q = , p 2 q3 = i
.
9
54
54
u=3
r2 =

7
1323 3 7
1323 1 3 7
1323 1 3 7
1323
+i
+
i
=
+i
+
i
.
54
54
54
54
3 2
2
3 2
2

49 1323 1372
+
=
= 343 , r = 343 = 7 7 , 3 r = 7 .
4
4
4

28

cos() =

7
7
1
=
=
= 0.188982236 , = 79.10660535 ,
2r 14 7 2 7

= 26.36886845, cos = 0.895953219 ,


3
3

4
1
u = 2 3 r cos = 1.580312937 , x = u = 0.246979603 .
3
3
3

Megkaptuk teht az xnket, ebbl h = 2.246979603 a hossz tl.

3. bra
A rvid tl meghatrozshoz a 3. bra kt fekete hasonl hromszgt
hvjuk segtsgl:
x
1
+
x
+
a
1
1+ x + y
1 + x = x 2 + 3x + 1 = 1.801937736 .
=
=
, a=
x
1+ x + y 1 y
1 y
1
1+ x
x kzelt meghatrozsa:
x3 + 4x 2 + 3x 1 = 0 , most elhanyagoljuk az x3 t:
9 + 4 4 1 3 5 3 1
=
= = 0.25 ,
8
8
4

4x 2 + 3x 1 = 0 , ennek megoldsa x =
elg jl kzelti az x = 0.246979603at.
Most azt igazoljuk, hogy a2 = x + 3 !

29

a = x 2 + 3x + 1 , s

x3 = 1 3x 4x 2 , ezzel

a 2 = (x 2 + 3x + 1)2 = x 4 + 9x 2 + 1 + 6x 3 + 2x 2 + 6x =
= x 4 + 6x 3 + 11x 2 + 6x + 1 = x(1 3x 4x 2 ) + 6(1 3x 4x 2 ) + 11x 2 + 6x + 1 =
= x 3x 2 4(1 3x 4x 2 ) + 6(1 3x 4x 2 ) + 11x 2 + 6x + 1 =
= x 3x 2 4 + 12x + 16x 2 + 6 18x 24x 2 + 11x 2 + 6x + 1 = 3 + x .
Teht sikerlt igazolni az a2 = x + 3 kpletet.
Az x egy algebrai szm, ami azt jelenti, hogy gyke egy vges fokszm
polinomnak. Ha a polinom legmagasabb fokszm tagjnak az egytthatja 1,
akkor xet algebrai egsznek nevezzk.
Az xbl az albbi algebrai egszek kpezhetk: z = a + bx + cx2 , ahol
a, b s c egsz szmok. Ezek az algebrai egszek gyrt kpeznek az
sszeadssal s a szorzssal mint mveletekkel. Ha megengedjk hogy az
a, b s c szmok racionlis szmok legyenek, s a gyr nullosztmentes,
akkor algebrai testet kapunk, amelyben elvgezhet az oszts minden nem
nulla szmmal. Mi most az algebrai egszek gyrjt nzzk meg rszletesen.
Legelszr is megalkotjuk az algebrai egszek mtrixreprezentcijt.
Ezt a kvetkezkppen tehetjk meg: vesszk a z, az xz s az x2z szmokat,
s ezek egytthatibl mint sorokbl mtrixot kpeznk.
A znek megfelel sorvektor az (a, b, c) sorvektor lesz.
xz = ax + bx2 + cx3 = ax + bx2 + c(1 3x 4x2) = c + (a 3c)x + (b 4c)x2 ,
ennek megfelelen a mtrix kzps sorvektora ez lesz: (c, a 3c, b 4c).

30

x2z = x(x2z) = cx + (a 3c)x2 + (b 4c) x3 =


= cx + (a 3c)x2 + (b 4c)(1 3x 4x2) =
= (b 4c) + ( 3b + 13c)x + (a 4b + 13c) x2 .
Ennek megfelelen a mtrix harmadik sorvektora ez lesz:
(b 4c , 3b + 13c , a 4b + 13c).
gy vgl ezt a mtrixot kapjuk:
a

b
c

a + bx + cx = c
a 3c
b 4c .
b 4c 3b + 13c a 4b + 13c

Ez nem ms, mint


1 0 0
0 1
0
0
0 1

a 0 1 0 + b 0 0 1 + c 1 3 4 = a 1 + b x + c x 2 .
0 0 1
1 3 4
4 13 13

Ha a b melletti mtrixot ngyzetre emeljk, a c melletti mtrixot kapjuk,


teht xx = x2 igaz rjuk. Ha pedig az xet az x2tel szorzom, kapom:
1
3 4

x x = x = 4 13 13 = 1 3x 4x 2 .
13 43 39

Most az xet reprezentl mtrix gy lett konstrulva, hogy kielgti az


x3 = 1 3x 4x2 harmadfok egyenletet. De ennek az egyenletnek
hrom gyke van! Most az gy kapott mtrix melyik gykt kpviseli?
Hamarosan vlaszt kapunk erre a krdsre is!

31

Most bevezetnk egy nagyon fontos fogalmat, a norma fogalmt.


A norma nem egyb, mint a z szmot reprezentl mtrix determinnsa.
A determinns pedig a mtrix 3 sorvektora ltal kifesztett paralelepipedon
trfogata. Ez a trfogat egsz szm, mgpedig annyi, ahny egsz rcspont esik
a paralelepipedon belsejbe, ha a 8 cscsot egynek vesszk (ezek algebrai
szempontbl ugyanazt a szmot reprezentljk) s ugyancsak egynek vesszk
a szemkzti oldalakra es rcspontokat, ha vannak ilyenek.
Ezek a paralelepipedonba es rcspontok adjk a z algebrai egsz ltal
ltestett maradkosztlyt, teht ha zvel maradkosan osztok, akkor a
lehetsges maradkokat. A determinns teht nem ms, mint a maradkosztly
elemeinek a szma. Illetve a determinns abszolt rtke, mert a determinns
lehet negatv is, st nulla is. Ekkor nullosztrl beszlnk.
Most szmoljuk ki ezt a determinnst!
D = a(a 3c)(a 4b + 13c) a(b 4c)( 3b + 13c) bc(a 4b + 13c) +
+ b(b 4c)(b 4c) + c2(3b + 13c) c(a 3c)(b 4c) =
= a3 4 a2b + 13 a2c 3 a2c + 12 abc 39 ac2 + 3 ab2 13 abc 12 abc +
+ 52 ac2 abc +4 b2c 13 bc2 + b3 4b2c 4b2c + 16 bc2 3bc2 + 13 c3 abc +
+ 4 ac2 + 3 bc2 12 c3 :
D = a3 + b3 + c3 4 a2b + 3 ab2 + 10 a2c + 17 ac2 4 b2c + 3 bc2 15 abc
Ezt a szmot gy jelljk: D = N(a, b, c). Ez a norma teht.

32

Most ellenrzskppen szmoljuk ki nhny z szm normjt!


z = 1 + x : N(1 + x) = N(1, 1, 0) = 1 + 1 + 0 4 + 3 + 0 + 0 0 + 0 0 = 1.
Ha a norma 1 vagy 1, akkor a szmot gyregysgnek nevezzk.
A gyregysgek a szorzsra nzve csoportot alkotnak, kt gyregysg
szorzata szintn gyregysg. A norma legfontosabb tulajdonsga az, hogy
szorzattart, azaz kt szm szorzatnak normja = a normk szorzatval!
Ennek beltshoz elg azt tudni, hogy a determinns maga is szorzattart!
w = 1 + x2 : N(1 + x2) = N(1, 0, 1) = 1 + 0 + 1 0 + 0 + 10 + 17 0 + 0 0 = 29
Mennyi N(zw) ? A vrt eredmny N(z) N(w) = 1 29 = 29 kell legyen.
zw = (1 + x)(1 + x2) = 1 + x + x2 + x3 = 1 + x + x2 + 1 3x 4x2 = 2 2x 3x2
zw = (2, 2, 3).
N(2, 2, 3) = 8 8 27 + 442 + 324 1043 + 1729 + 443
329 15223 =
= 8 8 27 + 32 + 24 120 + 306 + 48 54 180 = 29.
D = a3 + b3 + c3 4 a2b + 3 ab2 + 10 a2c + 17 ac2 4 b2c + 3 bc2 15 abc
teht jnak tnik. Bevallom, mikor ezt kiszmoltam, elszr egy szmolsi
hiba miatt N(1, 2, 3)at szmoltam, ami 13nak addott, sehogy se rtettem,
mirt nem stimmel az eredmny. Ezrt fontos az ellenrzs!
Miutn megismerkedtnk a normval, szmoljuk ki a mr ismert
(x 1)(x 2)(x + 3) = 0 egyenletnk gykeit reprezentl mtrixot!
Itt x3 = 7x 6. Meg fogunk lepdni az eredmnyen, s j felismersekre jutunk!

33

z = a + bx + cx2 , ennek megfelelen a mtrix els sora (a, b, c) lesz.


xz = ax + bx2 + cx3 = ax + bx2 + c(7x 6) = 6c + (a + 7c)x + bx2 ,
ennek megfelelen a mtrix msodik sora ( 6c, a + 7c, b) lesz,
x2z = x(xz) = x( 6c + (a + 7c)x + bx2 ) = 6cx + (a + 7c)x2 + bx3 =
= 6cx + (a + 7c)x2 + b(7x 6) = 6b + ( 6c + 7b)x + (a + 7c)x3 ,
ennek megfelelen a mtrix 3. sora ( 6b, 6c + 7b, a + 7c) lesz.
A mtrixunk teht:
a
b
c

z = 6c a + 7c
b .
6b 6c + 7b a + 7c

Innen lthat, hogy


0 1 0
0
0 1

2
x = 0 0 1 , x = 6 7 0 .
6 7 0
0 6 7

Az is lthat, hogy det x = 6, det x2 = 36.


Egyszer szmols meggyz arrl, hogy xx2 = x3 = 7x 6 teljesl.
Mennyi a z mtrix determinnsa?
N(a, b, c) = D = a(a + 7c)(a + 7c) ab(6c + 7b) + 6bc(a + 7c) 6b3
6c2(6c +7b) + 6bc(a + 7c) =
= a3 + 49 ac2 + 14 a2c + 6 abc 7ab2 + 6 abc + 42 bc2 6 b3 + 36 c3
42 bc2 + 6 abc + 42 bc2 :
N(a, b, c) = a3 6 b3 + 36 c3 7ab2 + 49 ac2 + 14 a2c + 42 bc2 + 18 abc .
Vegynk nhny konkrt pldt!

34

N(1, 1, 1) = 1 6 + 36 7 + 49 + 14 + 42 + 18 = 147 = 377.


Nemsokra ltni fogjuk, hogy ennek a szorzatrabontsnak jelentsge van!
N(1, 1, 0) = 1 6 + 0 7 + 0 + 0 + 0 + 0 = 12 = 23(2).
N(0, 1, 1) = 0 6 + 36 0 + 0 + 0 + 42 + 0 = 72 = 266.
(1, 1, 0)2 = (1 + x)2 = 1 + 2x + x2 = (1, 2, 1).
Mivel N(1, 1, 0) = 12 volt, N(1, 2, 1) = 144 kell hogy legyen.
N(1, 2, 1) = 1 68 + 36 74 + 49 + 14 + 422 + 182 = 144, ok!
N(1, 1, 0) = 1 + 6 7 = 0 , hopp, itt nullosztk is vannak!
(1, 1, 0)(1, 1, 0) = (1 + x)(1 x) = 1 x2 = (1, 0, 1) ,
N(1, 0, 1) = 0 kell teht legyen!
N(1, 0, 1) = 1 36 +49 14 = 0 , valban!
Amikor N(1 x) = 0 volt, akkor az x gy viselkedett, mintha x = 1 lenne!
s most idzzk emlkezetnkbe az x3 = 7x 6 egyenlet gykeit:
x1 = 1, x2 = 2, x3 = 3 !
Vajon ha az x gy viselkedik, mint a 2, vagy a 3, akkor is 0 lesz a norma?
N(2 x) = N(2, 1, 0) = 8 + 6 72 = 0 valban!
N(3 + x) = N(3, 1, 0) = 27 6 73 = 0 ! remnyeinkben nem csaldtunk!
z = a + bx + cx2 mi lesz, ha x = 1, x = 2 vagy x = 3? Nem ms, mint
z1 = a + b + c, z2 = a + 2b + 4c, s z3 = a 3b + 9c !
Lehet, hogy a norma ebbl a 3 faktorbl tevdik ssze? Nzzk meg!

35

z1 z2 z3 = (a + b + c) (a + 2b + 4c) (a 3b + 9c) =
(a2 + 2ab + 4ac + ab + 2b2 + 4bc + ac + 2bc + 4c2 ) (a 3b + 9c) =
= (a2 + 3ab + 5ac + 2b2 + 6bc + 4c2) (a 3b + 9c) =
= a3 + 3 a2b + 5 a2c + 2 ab2 + 6 abc + 4 ac2 3a2b 9 ab2 15 abc
6b3 18 b2c 12 bc2 + 9 a2c + 27 abc + 45 ac2 + 18 b2c + 54 bc2 + 36 c3 =
= a3 + 14 a2c 7 ab2 + 18 abc + 49 ac2 6b3 + 42 bc2 + 36 c3 = N(a, b, c) !
Ltjuk, a sorrendtl eltekintve ugyanazt kaptuk valban!
Ezzel egy nagy felismersre jutottunk:
A normakifejezs ltalnos alakja:
N(a, b, c) = (a + bx1 + cx12)( a + bx2 + cx22)( a + bx3 + cx32) !
A z1 = a + bx1 + cx12 algebrai egsz konjugltjainak nevezzk a
z2 = a + bx2 + cx22 algebrai egszt s a z3 = a + bx3 + cx32 algebrai egszt,
ahol x1, x2, s x3 ugyanannak az egyenletnek a gykei.
Ha a fenti normakifejezst sszeszorozzuk, ilyen tagokat kapunk:
a3 , b3(x1x2x3) , ab2(x1x2 + x1x3 + x2x3 ) , a2b(x1 + x2 + x3) , s itt
rgtn felismerhetjk a gykk s egytthatk sszefggseit!
Ha x3 + px2 + qx + s = 0 az egyenletnk, s ezt ilyen alakban rjuk:
(x x1)( x x2)( x x3) = 0, akkor azt ltjuk, hogy
x2 egytthatja (x1 + x2 + x3 ) = p,
x egytthatja (x1x2 + x1x3 + x2x3) = q, s
a nulladfok tag x1x2x3 = s lesz, azaz nlunk x3 7x + 6 = 0 miatt
x1 + x2 + x3 = 0, mert nincs msodfok tag, ezrt az a2b egytthatja a normban
36

nulla lesz, s lthatjuk, hogy tnyleg nincs benne a2b s tag!


Az ab2es tag egytthatja (x1x2 + x1x3 + x2x3) = q = 7 , s tnyleg az!
A b2c egytthatja (x1x2x32 + x1x22x3 + x12x2x3 ) = x1x2x3(x1 + x2 + x3) = sp
azaz 0, mert nlunk p = 0, s lthatjuk, hogy tnyleg nincs b2c a normban!
A bc2 egytthatja (x1x22x32 + x12x2x32 + x12x22x3 ) =
= x1x2x3(x1x2 + x1x3 + x2x3) = sq = 67 = 42, s tnyleg annyi!
Vannak olyan tagok, amiket egyenlre nem tudunk rtelmezni:
Az abc egytthatja (x1x22 + x1x32 + x2x32 + x12x2 + x12x3 + x22x3),
ezt most nem tudjuk a gykk s egytthatk sszefggseivel megadni.
Az ac2 egytthatja (x12x22 + x12x32 + x22x32), ezt se tudjuk megadni.
Csinljunk egy trkkt: ha valami nulla, akkor a ngyzete is nulla!
Teht ha (x x1)( x x2)( x x3) = 0, akkor (x x1) 2( x x2) 2( x x3) 2 = 0!
Kifejtve: (x2 + x12 2xx1) (x2 + x22 2xx2) (x2 + x32 2xx3) = 0.
Most ha sszeszorozzuk, akkor pl. az x4 egytthatja
(x12 + x22 + x32) + 4 (x1x2 + x1x3 + x2x3) lesz!
(x1x2 + x1x3 + x2x3) = q = 7 , s ha az egyenletet ngyzetre emelem:
(x3 7x + 6) 2 = x6 + 49 x2 + 36 14 x4 + 12 x3 84 x,
az x4es tag egytthatja 14 = (x12x22 + x12x32 + x22x32) 28 teht,
teht (x12 + x22 + x32) = 14 kell legyen. Ez, ha megnzzk, az a2c s tag
egytthatja, s az valban 14!

37

A gykkbl kpzett szimmetrikus polinomokkal teht ki tudjuk fejezni


a normt! A Galoiselmlet ppen a szimmetrikus polinomokra pl.
Most kivtelesen ismerjk magukat a gykket is, s azokkal kzvetlenl is
ki tudjuk szmolni a szimmetrikus hatvnysszegeket:
x1 = 1, x2 = 2, x3 = 3, ezekkel
(x12 + x22 + x32) = 1 + 4 + 9 = 14,
(x12x22 + x12x32 + x22x32) = 14 + 19 + 49 = 49,
x12x22x32 = 149 = 36.
Ezekkel meg tudjuk adni azt az egyenletet, amelynek a gykei x 12, x22, x32 !
Ez az egyenlet gy ll el: (x x12) (x x22) (x x32) = 0, azaz
x3 (x12 + x22 + x32) x2 + (x12x22 + x12x32 + x22x32) x x12x22x32 = 0
ebbl x3 14 x2 + 49 x 36 = 0 addik.
Az xet s az x2et reprezentl mtrixokbl is megkapjuk az egyenletet:
A mtrix karakterisztikus egyenlete gy nz ki: det (A E) = 0, ahol
a sajtrtk, s E a harmadrend egysgmtrix.
Tegyk az A helybe az x mtrixot:
1
0

det(x E) = det 0 1 = 3 + 7 6 = 0 , teht 3 7 + 6 = 0 !


6 7

Most mr vlaszt tudunk adni arra a krdsre, hogy melyik gykt kpviseli
az x mtrix: mind a hrmat! Lttuk, hogy a norma kifejezsben mind a 3
gyk szerepel! Teht az x mtrix gy viselkedik, mint a 3 gyk egyttese!

38


0
1
2

det(x E) = det 6 7 0 = (7 ) + 36 = 0 , teht


0
6 7

3 + 14 2 49 + 36 = 0 , azaz 3 14 2 + 49 36 = 0 .
Megkaptuk teht az egyenletnket.
Most rjuk fel a normt a szimmetrikus polinomokkal!
N(a, b, c) = (a + bx1 + cx12)( a + bx2 + cx22)( a + bx3 + cx32) =
= a3 + x1x2x3 b3 + x12x22x32 c3 + a2b(x1 + x2 + x3 ) +
+ ab2(x1x2 + x1x3 + x2x3) + a2c(x12 + x22 + x32 ) +
+ ac2(x12x22 + x12x32 + x22x32) + b2c(x1x2x32 + x1x22x3 + x12x2x3 ) +
+ bc2(x1x22x32 + x12x2x32 + x12x22x3 ) +
+ abc(x1x22 + x1x32 + x2x32 + x12x2 + x12x3 + x22x3).
Ha x3 + px2 + qx + s = 0 az egyenletnk, akkor (x3 + px2 + qx + s) 2 = 0,
azaz x6 + p2x4 + q2x2 + s2 + 2px5 + 2qx4 + 2sx3 + 2pqx3 + 2psx2 + 2qsx =0,
rendezve:
x6 + 2px5 + (p2 + 2q) x4 + (2s + 2pq) x3 + (2ps + q2) x2 + 2qsx + s2 = 0.
(x x1) 2( x x2) 2( x x3) 2 = 0:
(x2 + x12 2xx1) (x2 + x22 2xx2) (x2 + x32 2xx3) = 0.

Kifejtve:

x6 2 (x1 + x2 + x3 ) x5 + (x12 + x22 + x32 + 4 x1 x2 + 4 x1 x3 + 4 x2 x3) x4


(2(x12 x2 + x12 x3 + x22 x3 + x1 x22 + x1 x32 + x2 x32 )+ 8 x1 x2 x3 ) x3 +
+ (x12 x22 + x12 x32 + x22 x32 + 4(x1x2x32 + x1x22x3 + x12x2x3)) x2
2 (x1x22x32 + x12x2x32 + x12x22x3 ) x + x12 x22 x32 = 0.

39

sszehasonltva az egytthatkat, ezt kapjuk:


p = (x1 + x2 + x3 ), ezt eddig is tudtuk.
p2 + 2q = x12 + x22 + x32 + 4 x1 x2 + 4 x1 x3 + 4 x2 x3 = x12 + x22 + x32 + 4 q,
amibl x12 + x22 + x32 = p2 2q . Nlunk ez 14.
2s + 2pq = 2(x12 x2 + x12 x3 + x22 x3 + x1 x22 + x1 x32 + x2 x32) 8 x1 x2 x3, ebbl
x12 x2 + x12 x3 + x22 x3 + x1 x22 + x1 x32 + x2 x32 = 3s pq , nlunk 18.
Valban, 12 13 43 + 14 + 19 + 29 = 18.
2ps + q2 = x12 x22 + x12 x32 + x22 x32 + 4(x1x2x32 + x1x22x3 + x12x2x3), azaz
2ps + q2 = x12 x22 + x12 x32 + x22 x32 + 4ps, ebbl
x12 x22 + x12 x32 + x22 x32 = q2 2ps, nlunk 49.
2qs = 2 (x1x22x32 + x12x2x32 + x12x22x3 ), ez is trivialits.
s2 = x12 x22 x32 , ez is trivialits.
Megkaptuk teht a minket rdekl szimmetrikus hatvnysszegeket.
Ha teht z = a + bx + cx2 , s x kielgti az x3 + px2 + qx + s = 0 egyenletet,
akkor N(z) = N(a, b, c) = a3 s b3 + s2 c3 p a2b + q ab2 + ( p2 2q) a2c +
+ (q2 2ps) ac2 + ps b2c qs bc2 + (3s pq) abc lesz a norma.
Legyen most x3 = 1 3x 4x2 az egyenletnk. Mint lttuk, a norma ekkor
N = a3 + b3 + c3 4 a2b + 3 ab2 + 10 a2c + 17 ac2 4 b2c + 3 bc2 15 abc.
Ellenrizzk a kpletnket!
p = 4, q = 3, s = 1, teht
N = a3 + b3 + c3 4 a2b + 3 ab2 + (16 6) a2c + (9 + 8) ac2 4 b2c +
+ 3 bc2 + (3 12) abc, s tnyleg stimmel!
40

Mint ltjuk, a kpleteink akkor is stimmelnek, ha a gykk irracionlisak.


Akkor is stimmelnek, ha komplex gykk is vannak!
Trjnk vissza a bevezetben emltett sorozathoz!
1, 1, 1, 2, 2, 3, 4, 5, 7, 9, 12, 16, 21, 28, 37, 49, 65, 86,
114, 151, 200, 265, 351, 465, 616, 816, 1081, 1432, 1897,
2513, 3329, 4410, 5842, 7739, 10252, 13581, 17991, 23833,
31572, 41824

Ennl kt szomszdos szm hnyadosa ahhoz az x szmhoz tart, amely kielgti


1
23 3 1
23
az x3 x 1 = 0 egyenletet. x = 3 +
+
= 1.324717957
2 108
2 108
Ez egy vals gyk, s mellette mg van kt konjuglt komplex gyk is.
Mennyi itt a z = a + bx + cx2 szm normja?
p = 0, q = 1, s = 1 , teht
N = a3 s b3 + s2 c3 p a2b + q ab2 + ( p2 2q) a2c +
+ (q2 2ps) ac2 + ps b2c qs bc2 + (3s pq) abc :
N = a3 + b3 + c3 ab2 + 2 a2c + ac2 bc2 3 abc ,
nzzk meg, jl szmoltunk e?
N(1, 1, 0) = 1 + 1 1 =1, na, talltunk egy egysget!
(1, 1, 0)2 = (1, 2, 1),
N(1, 2, 1) = 1 + 8 + 1 4 + 2 + 1 2 6 = 1, ez teht rendben van.
Az egysg minden hatvnya egysg, ezrt ebben a gyrben vgtelen sok
egysg van! Nzzk meg az egysg hatvnyait!
(1, 1, 0)3 = 1 + 3x + 3x2 + x3 = 2 + 4x + 3x2 = (2, 4, 3)
(1, 1, 0)4 = 2 + 4x + 3x2 + 2x + 4x2 + 3x3 = (5, 7, 7)
41

(1, 1, 0)5 = (5, 7, 7) + (0, 5, 7) + (7, 7, 0) = (12, 19, 14)


(1, 1, 0)6 = (12, 19, 14) + (0, 12, 19) + (14, 14, 0) = (26, 45, 33)
N(26, 45, 33) = 17576 + 91125 + 35937 262025 +267633 + 261089
451089 3264533 = 83, ennek 1nek kne lennie!
N = a3 + b3 + c3 ab2 + 2 a2c + ac2 bc2 3 abc ,
N(5, 7, 7) = 125 + 343 + 343 549 + 2257 + 549 749 3577 = 83
teht mr itt hiba van!
(1, 1, 0)4 = 2 + 4x + 3x2 + 2x + 4x2 + 3x3 = (5, 9, 7) !!!
N(5, 9, 7) = 125 + 729 + 343 581 + 2257 + 549 949 3597 = 1.
Szndkosan hagytam benne ezt a hibt, hogy prezentljam, hogyan szokott
az ember tveszteni, s hogyan tudja a hibt lebuktatni!
Az elvrsom az volt, hogy az (a, b, c) szmban az a helyn ll szm ppen
a sorozatunk eleme legyen, s a 26 kilgott, itt fogtam gyant!
(1, 1, 0)5 = (5, 9, 7) + (0, 5, 9) + (7, 7, 0) = (12, 21, 16)
(1, 1, 0)6 = (12, 21, 16) + (0, 12, 21) + (16, 16, 0) = (28, 49, 37)
(1, 1, 0)7 = (28, 49, 37) + (0, 28, 49) + (37, 37, 0) = (65, 114, 86)
Ltjuk, hogy az algebrai szmunk els tagja, az a az albbi sorozatot
alkotja: 1, 1, 2, 5, 12, 28, 65, = a[1], a[2], a[5], a[8], a[11], a[14], a[17],
itt feltettk, hogy a[0] = 0, s gy a[1] = 1.
Az algebrai szm msodik tagja, a b szintn egy sorozatot alkot:
1, 2, 4, 9, 21, 49, 114, itt 114 = 249 + 29 2 , 49 = 221 + 24 1,
azt vrjuk, hogy ez a sorozat is engedelmeskedik valami szablynak.
42

Sorozatokkal kezdtk ezt a cikket, s sorozatokkal fejezzk is be.


Ha jobban rdekelnek a sorozatok, a Sloane katalgusban tbb mint szzezret
tallhatsz. rd be a Google-ba azt hogy sloanes on-line, s az els tallatot vedd.
Itt a keresbe be lehet rni egy szmsorozat els nhny tagjt, s kiadja hogy
van e ilyen a katalgusban. Ha nincs, s tudod, hogyan kell generlni, akkor
berhatod mint j sorozatot, s ha elfogadjk, akkor a te neveden fut!
Ha berod a Sloane keresjbe azt hogy kristmikl, kiadja az n sorozataimat.
Ht mr tbb mint szzat rtam be!
Vannak harmadfok sorozatok is, azaz olyan sorozatok, amelyeknl a kt
egymst kvet elem hnyadosa egy harmadfok egyenletnek engedelmesked
szmhoz tart. Nhny plda ilyenre:
a[1]:=1:a[2]:=1:a[3]:=1:for n from 4 to 30 do
a[n]:=a[n-1]+a[n-3] od:
seq(a[n],n=1..30);
1, 1, 1, 2, 3, 4, 6, 9, 13, 19, 28, 41, 60, 88, 129, 189, 277,
406, 595, 872, 1278, 1873, 2745, 4023, 5896, 8641, 12664,
18560, 27201, 39865
a[1]:=1:a[2]:=1:a[3]:=1:for n from 4 to 30 do
a[n]:=a[n-1]+2*a[n-3] od:
seq(a[n],n=1..30);
1, 1, 1, 3, 5, 7, 13, 23, 37, 63, 109, 183, 309, 527, 893,
1511, 2565, 4351, 7373, 12503, 21205, 35951, 60957, 103367,
175269, 297183, 503917, 854455, 1448821, 2456655
a[1]:=1:a[2]:=1:a[3]:=1:for n from 4 to 30 do
a[n]:=a[n-2]+2*a[n-3] od:
seq(a[n],n=1..30);
1, 1, 1, 3, 3, 5, 9, 11, 19, 29, 41, 67, 99, 149, 233, 347,
531, 813, 1225, 1875, 2851, 4325, 6601, 10027, 15251, 23229,
35305, 53731, 81763, 124341

43

Most jvk csak r, hogy egy kicsit figyelmetlen voltam. Az algebrai szm
msodik eleme ltal alkotott 1, 2, 4, 9, 21, 49, 114, sorozat nem egyb, mint
a[1], a[4], a[7], a[10], a[13], a[16], a[19], ! s a harmadik elem ltal alkotott
0, 1, 3, 7, 16, 37, 86, 200, sorozat nem egyb, mint a[0], a[3], a[6], a[9], !
Itt a[0] = 0 vlasztssal ltnk.
De adjuk meg a sorozatot gy:
a[1]:=1:a[2]:=0:a[3]:=0:for n from 4 to 40 do
a[n]:=a[n-2]+a[n-3] od:
seq(a[n],n=1..40);
1, 0, 0, 1, 0, 1, 1, 1, 2, 2, 3, 4, 5, 7, 9, 12, 16, 21, 28,
37, 49, 65, 86, 114, 151, 200, 265, 351, 465, 616, 816, 1081,
1432, 1897, 2513, 3329, 4410, 5842, 7739, 10252

Ekkor mondhatjuk:
(1 + x)n = (1, 1, 0)n = a[3n + 1] + a[3n + 3]x + a[3n + 2]x2 , n = 0, 1, 2,
gy valban az albbi hatvnysort kapjuk:
(1, 1, 0)0 = (1, 0, 0)
(1, 1, 0)1 = (1, 1, 0)
(1, 1, 0)2 = (1, 2, 1)
(1, 1, 0)3 = (2, 4, 3)
(1, 1, 0)4 = (5, 9, 7)
Na, most az a krds, hogy akkor az x egyenletnek 3 gykvel ltre lehet
hozni magt a sorozatot is? A vlasz az, hogy igen!
Elszr szmoljuk ki a sorozat genertorfggvnyt!

44

A genertorfggvny: g(x) = a[0] + a[1]x + a[2]x2 + a[3]x3 + a[4]x4 +


Mennyi a[0] ? Nos, a[0] + a[1] = a[3], azaz a[0] + 1 = 0, teht a[0] = 1.
g(x) + xg(x) = a[0] + (a[0]+ a[1])x + (a[1]+ a[2])x2 + (a[2]+ a[3])x3 + =
= a[0] + a[3]x + a[4]x2 + a[5]x3 + a[6]x4 + =
g(x)(x3+x2 1) = 1 x x2, innen g(x) =

g(x) a 0 a 1 x
+ a 0 ,
x2

1 x x2
.
1 + x 2 + x 3

Most ezt leteszteljk a Maple 7 programmal:


restart:
G(x):=(1-x-x^2)/(-1+x^2+x^3):
f[0]:=G(x):
for n from 1 to 30 do f[n]:=diff(f[n-1],x) od:
x:=0:
seq(f[n]/n!,n=0..30);
-1, 1, 0, 0, 1, 0, 1, 1, 1, 2, 2, 3, 4, 5, 7, 9, 12, 16, 21,
28, 37, 49, 65, 86, 114, 151, 200, 265, 351, 465, 616

Nagyon j, megkaptuk a sorozatunkat, mg a[0] is megvan az elejn!


A sorozat gy szerepel a Sloane katalgusban:
A000931 Padovan sequence: a(n) = a(n-2) + a(n-3).
(Formerly M0284 N0102)
1, 0, 0, 1, 0, 1, 1, 1, 2, 2, 3, 4, 5, 7, 9, 12, 16, 21, 28, 37, 49, 65, 86, 114, 151,
200, 265, 351, 465, 616, 816, 1081, 1432, 1897, 2513, 3329, 4410, 5842, 7739,
10252, 13581, 17991, 23833, 31572, 41824, 55405, 73396, 97229, 128801,
170625
FORMULA G.f.: (1-x^2)/(1-x^2-x^3).
a(n) is asymptotic to r^n / (2*r+3) where r = 1.3247179572447..., the real root of
x^3 = x + 1 . - DELEHAM Philippe (kolotoko(AT)wanadoo.fr), Jan 13 2004
a(n)^2+a(n+2)^2+a(n+6)^2 = a(n+1)^2+a(n+3)^2+a(n+4)^2+a(n+5)^2
(Barniville, Question 16884, Ed. Times 1911).
45

a(n) = central and lower right terms in the (n-3)-th power of the 3 X 3 matrix
M = [0 1 0 / 0 0 1 / 1 1 0]. E.g. a(13) = 7. M^10 = [3 5 4 / 4 7 5 / 5 9 7].
- Gary W. Adamson (qntmpkt(AT)yahoogroups.com), Feb 01 2004
G.f.: 1/(1-x^3-x^5-x^7-x^9-....) - Jon Perry (perry(AT)globalnet.co.uk),
Jul 04 2004
a(n+4)=sum{k=0..floor((n-1)/2), binomial(floor((n+k-2)/3), k)}.
- Paul Barry (pbarry(AT)wit.ie), Jul 06 2004
a(n)=sum{k=0..floor(n/2), binomial(k, n-2k)} - Paul Barry (pbarry(AT)wit.ie),
Sep 17 2004
a(n+3) is diagonal sum of A026729 (as a number triangle), with formula
a(n+3)=sum{k=0..floor(n/2), sum{i=0..n-k, (-1)^(n-k+i)C(n-k, i)C(i+k, i-k)}}
- Paul Barry (pbarry(AT)wit.ie), Sep 23 2004
a(n) = a(n-1)+a(n-5) = A003520(n-4)+A003520(n-13) =
= A003520(n-3)-A003520(n-9).
- Henry Bottomley (se16(AT)btinternet.com), Jan 30 2005
a(n+3)=sum{k=0..floor(n/2), C((n-k)/2, k)(1+(-1)^(n-k))/2};
- Paul Barry (pbarry(AT)wit.ie), Sep 09 2005
The sequence 1/(1-x^2-x^3) (a(n+3)) is given by the diagonal sums of the
Riordan array (1/(1-x^3), x/(1-x^3)). The row sums are A000930.
- Paul Barry (pbarry(AT)wit.ie), Feb 25 2005
a(n) = A023434(n-7)+1 for n>=7. - David Callan (callan(AT)stat.wisc.edu),
Jul 14 2006
a(n+5) corresponds to the diagonal sums of A030528. The binomial transform of
a(n+5) is A052921.
a(n+5)=
=sum{k=0..floor(n/2), sum{k=0..n, (-1)^(n-k+i)C(n-k, i)C(i+k+1, 2k+1)}}.
- Paul Barry (pbarry(AT)wit.ie), Jun 21 2004
r^(n-1) = (1/r)*a(n) + r*a(n+1) + a(n+2); where r = 1.32471... is the real root of
x^3 - x - 1 = 0. Example: r^8 = (1/r)*a(9) + r*a(10) + a(11) = (1/r)*2 + r*3 + 4 =
=9.483909... - Gary W. Adamson (qntmpkt(AT)yahoo.com), Oct 22 2006

46

a(n) = (r^n)/(2r+3) + (s^n)/(2s+3) + (t^n)/(2t+3) where r, s, t are the three roots


of x^3-x-1 in any order. - Keith Schneider (schneidk(AT)email.unc.edu),
Sep 07 2007
EXAMPLE When run backwards gives (-1)^n*A050935(n).
MAPLE A000931 := proc(n) option remember; if n = 0 then 1 elif n <= 2
then 0 else A000931(n-2)+A000931(n-3); fi; end;
A000931:=-(1+z)/(-1+z**2+z**3);
[Conjectured by S. Plouffe in his 1992 dissertation.]
MATHEMATICA
CoefficientList[Series[(1 - x^2)/(1 - x^2 - x^3), {x, 0, 50}], x]
a[0] = 1; a[1] = a[2] = 0; a[n_] := a[n] = a[n - 2] + a[n - 3]; Table[a[n], {n, 0,
51}] (from Robert G. Wilson v (rgwv(at)rgwv.com), May 04 2006)
Na, ez van a Sloane katalgusban, sok rdekeset tudhatunk meg belle.
Kln kiemelem a mtrixra vonatkoz megjegyzst:
a(n) = central and lower right terms in the (n-3)-th power of the 3 X 3 matrix
M = [0 1 0 / 0 0 1 / 1 1 0]. E.g. a(13) = 7. M^10 = [3 5 4 / 4 7 5 / 5 9 7].
0 1 0

Itt a 0 0 1 mtrix van emlegetve, ami nem egyb, mint az x3 = 1 + x


1 1 0

10

0 1 0

egyenletet kielgt gykt reprezentl mtrix. 0 0 1


1 1 0

3 5 4

= 4 7 5 .
5 9 7

Most legyen x1, x2, x3 az x3 = 1 + x egyenlet 3 gyke! (kett konjuglt komplex)


Nzzk, milyen sorozatot kapunk az a[n] = x1 n + x2 n + x3 n hatvnysszegbl!
a[0] = 1 + 1 + 1 = 3, a[1] = x1 + x2 + x3 = 0, mert nincs msodfok tag.

47

a[2] = x1 2 + x2 2 + x3 2 = (u + v) 2 + (u + 2v) 2 + (2u + v) 2 =


= u2 + v2 + 2uv + 2u2 + 4v2 + 2u2v + 4u2 + 2v2 + 22uv =
= u2 + v2 + 2uv + 2u2 + v2 + 2uv + u2 + 2v2 + 2uv = 6uv = 2,
mert 3 = 1, 3uv = 3 3 a 2 b 2 = 1 a megoldkplet alapjn.
Teht a sorozatunk gy indul: 3, 0, 2,
sejtsnk az, hogy gy fog folytatdni: a[n] = a[n 2] + a[n 3],
teht a sorozatunk: 3, 0, 2, 3, 2, 5, 5, 7, 10, 12, 17, 22, 29,
Figyeljk meg, hogy az a[n] = a[n 1] + a[n 5] szably is teljesl!
Rkerestem a Sloane katalgusban:
A001608 Perrin (or Ondrej Such) sequence: a(n) = a(n-2) + a(n-3).
(Formerly M0429 N0163)
3, 0, 2, 3, 2, 5, 5, 7, 10, 12, 17, 22, 29, 39, 51, 68, 90, 119, 158, 209, 277, 367,
486, 644, 853, 1130, 1497, 1983, 2627, 3480, 4610, 6107, 8090, 10717, 14197,
18807, 24914, 33004, 43721, 57918, 76725, 101639, 134643, 178364, 236282,
313007
Asymptotically, a(n) ~ r^n, with r=1.3247179572447... the inverse of the real
root of 1-x^2-x^3=0
FORMULA G.f.: (3 - x^2)/(1 - x^2 - x^3).
a(n)=r1^n+r2^n+r3^n where r1, r2, r3 are three roots of x^3-x-1=0.
a(n-1)+a(n)+a(n+1)=a(n+4), a(n)-a(n-1)=a(n-5). - Jon Perry
(perry(AT)globalnet.co.uk), Jun 05 2003
a(n) = the trace of M^n where M = the 3 X 3 matrix [0 1 0 / 0 0 1 / 1 1 0].
2. a(n) = 2*A000931(n+3) + A000931(n)
E.g. a(10) = 17 = (3 + 7 + 7) = trace of M^10. - Gary W. Adamson
(qntmpkt(AT)yahoo.com), Feb 01 2004

48

MAPLE A001608:=-z*(2+3*z)/(-1+z**2+z**3);
[Conjectured by S. Plouffe in his 1992 dissertation.]
PROGRAM (PARI) a(n)=if(n<0, 0, polsym(x^3-x-1, n)[n+1])
A kkkel kiemelt kt sor utal arra, hogy a[n] = x1 n + x2 n + x3 n , illetve a mr
emltett reprezentl mtrix sprja, azaz a ftlban lev elemek sszege.
Vgl rkerestem arra a szra hogy Tribonacci sequence. Ez jtt be:
A000073 Tribonacci numbers: a(n) = a(n-1) + a(n-2) + a(n-3).
(Formerly M1074 N0406)
0, 0, 1, 1, 2, 4, 7, 13, 24, 44, 81, 149, 274, 504, 927, 1705, 3136, 5768, 10609,
19513, 35890, 66012, 121415, 223317, 410744, 755476, 1389537, 2555757,
4700770, 8646064, 15902591, 29249425, 53798080, 98950096, 181997601,
334745777
A000213 Tribonacci numbers: a(n) = a(n-1) + a(n-2) + a(n-3).
(Formerly M2454 N0975)
1, 1, 1, 3, 5, 9, 17, 31, 57, 105, 193, 355, 653, 1201, 2209, 4063, 7473, 13745,
25281, 46499, 85525, 157305, 289329, 532159, 978793, 1800281, 3311233,
6090307, 11201821, 20603361, 37895489, 69700671, 128199521, 235795681,
433695873
a(n)/a(n-1) tends to the tribonacci constant, 1.839286755...; an eigenvalue of M
and a root of x^3 - x^2 - x - 1 = 0. - Gary W. Adamson
MAPLE K:=(1-z^2)/(1-z-z^2-z^3):
Kser:=series(K, z=0, 45): seq((coeff(Kser, z, n)), n= 0..34);
- Zerinvary Lajos (zerinvarylajos(AT)yahoo.com), Nov 08 2007
Ltjuk, hogy ez is harmadfok sorozat. Teht a tmnkba vg.
Felbukkan Zerinvry Lajos neve is, akit vekkel ezeltt n segtettem,
azta szorgalmasan ontja a sorozatokat.
Na, ht ennyit akartam elmondani, ksznm hogy elolvasttok.

49

You might also like